Answer:
Step-by-step explanation:
should be 1110
Answer:
1110
Step-by-step explanation:
In the data set below, what is the range? 9, 4, 9, 2, 8, 2, 1
EH is a diameter is D. The measure of EF is (10x + 8) and the measure of GH is (11x). What's the value of X
Answer:
x = 5°
EF = 58°
GH = 55°
Step-by-step explanation:
From The diagram :
Recall ; Angle on a straight line = 180°
This means :
EF + FG + GH = 180
EF = (10x + 8)°
GH = (11x)°
FG = 67°
HENCE;
10x + 8 + 11x + 67 = 180
21x + 75° = 180°
21x = 180 - 75
21x = 105
x = 105 / 21
x = 5
EF = 10x + 8 = 50 + 8 = 58°
GH = 11x = 11 * 5 = 55°
Regression analysis is better than the high-low method of cost estimation because regression analysis: Multiple Choice Fits data into a mathematical equation. Takes less time. Is mathematical. Can provide greater precision and reliability. Is a statistical method.
Answer:
hola no sé qué decirte no se Inglés
what is the surface area of this cylinder ?q
Answer:
130 pi or 408.41
Step-by-step explanation:
formula for sa of a cylinder is 2pir^2+2pirh
you just fill in the r and h
Answer:
\(\approx 408.2\) in²
Step-by-step explanation:
The surface area of a cylinder can be represented as \(2\pi rh+2\pi r2\). Let's represent pi as 3.14 and r is 5 and h is 8.
\((2\cdot 3.14\cdot 5\cdot8) + (2\cdot 3.14 \cdot 5^2)\\\\251.2 + 157 = 408.2\)
This means the surface area of this cylinder is 408.2 in².
Hope this helped!
Let X[k], k = 0, ..., M-1 be the DFT of M points of a real sequence x[n]. If we know the DFT value for a certain index k (0 < k < M-1), for what other index k2 ( 0< k2< M-1) can we determine the DFT value? What is the value of the DFT for k2?
If we know the DFT value for a certain index k (0 < k < M-1) of a real sequence x[n], we can determine the DFT value for another index k2 (0 < k2 < M-1) if k2 is related to k through complex conjugation. In other words, if k2 is the conjugate of k, then we can determine the DFT value for k2.
For a real sequence, the DFT values follow a symmetry property. If X[k] is the DFT value at index k, then X[M - k] is the DFT value at index k2, where k2 = M - k. The value of the DFT for k2 would be the complex conjugate of the DFT value for k, denoted as X[M - k] = X[k]*. The asterisk (*) represents complex conjugation.
In summary, if we know the DFT value for a certain index k in a real sequence, we can determine the DFT value for the index k2 = M - k, and the value of the DFT for k2 would be the complex conjugate of the DFT value for k.
Learn more about DFT value here: brainly.com/question/31473582
#SPJ11
Given a language L, the complement is defined as Lˉ={w∣w∈Σ∗ and w∈/L}. Given a language L, a DFA M that accepts L is minimal if there does not exist a DFA M′ such that M′ accepts L, and M′ has fewer states than M. (a) Prove that the class of regular languages is closed under complement. (b) Given a DFA M that accepts L, define Mˉ to be the DFA that accepts Lˉ using your construction from part (a). Prove that if M is minimal, then Mˉ is minimal.
If M is a minimal DFA accepting L, then the DFA Mˉ accepting the complement of L is also minimal.
(a) To prove that the class of regular languages is closed under complement, we need to show that for any regular language L, its complement Lˉ is also a regular language.
Let's assume that L is a regular language. This means that there exists a DFA (Deterministic Finite Automaton) M that accepts L. We need to construct a DFA M' that accepts the complement of L, Lˉ.
To construct M', we can simply swap the accepting and non-accepting states of M. In other words, for every state q in M, if q is an accepting state in M, then it will be a non-accepting state in M', and vice versa. The transition function and start state remain the same.
The intuition behind this construction is that M accepts strings that are in L, and M' will accept strings that are not in L. By swapping the accepting and non-accepting states, M' will accept the complement of L.
Since we can construct a DFA M' that accepts Lˉ from the DFA M that accepts L, we have shown that Lˉ is a regular language. Therefore, the class of regular languages is closed under complement.
(b) Now, let's assume that M is a minimal DFA that accepts the language L. We need to prove that Mˉ, the DFA accepting the complement of L, is also minimal.
To prove this, we can use a contradiction argument. Let's assume that Mˉ is not minimal, i.e., there exists a DFA M'' that accepts Lˉ and has fewer states than M. Our goal is to show that this assumption leads to a contradiction.
Since M is minimal, it means that there is no DFA M' that accepts L and has fewer states than M. However, we have assumed the existence of M'', which accepts Lˉ and has fewer states than M.
Now, consider the DFA M''', obtained by swapping the accepting and non-accepting states of M''. In other words, for every state q in M'', if q is an accepting state in M'', then it will be a non-accepting state in M''', and vice versa. The transition function and start state remain the same.
We can observe that M''' accepts L because it accepts the complement of Lˉ, which is L. Moreover, M''' has fewer states than M, which contradicts the assumption that M is minimal.
Therefore, our initial assumption that Mˉ is not minimal leads to a contradiction. Hence, if M is minimal, then Mˉ is also minimal.
In conclusion, we have proven that if M is a minimal DFA accepting L, then the DFA Mˉ accepting the complement of L is also minimal.
To know more about complement visit
https://brainly.com/question/29697356
#SPJ11
Suppose q is directly proportional to the square root of h and inversely proportional to the cube of j, and q=24 when h= 16 and j=2. Find q when h=9 and j=1/2
When h=9 and j=1/2 the q value is 1152.
Given that:
q is directly proportional to the square root of h and inversely proportional to the cube of j, and q=24 when h= 16 and j=2.
q ∝ \(\sqrt{h}\)/\(j^{3}\)
q = k \(\sqrt{h}\)/\(j^{3}\)
24 = k \(\sqrt{16}\)/\(2^{3}\)
24 = k 4/8
k = 24 *2
k = 48
q = 48 \(\sqrt{h}\)/\(j^{3}\)
substitute h = 9 and j = 1/2
q = 48\(\sqrt{9\)/\(1/2^{3}\)
q = 48 * 3/1/8
q = 48 * 3 * 8
q = 48 * 24
q = 1152
Therefore when h=9 and j=1/2 the q value is 1152.
Learn more about the directly proportional and square root here:
https://brainly.com/question/18557755
#SPJ1
One hundred pyramid-shaped chocolate candies with a square base with 12 mm sides and height of 15 mm are melted in a cylindrical pot. If the pot has a radius of 75 mm, what is the height of the melted candies in the pot?
Step-by-step explanation:
Volume of 100chocolates:
((12^2 x 15)/3) x 100 =72000
volume of cylinder:
base area x height
base area= πr^2= 5625π
height=volume/base area
height=72000/5625π=4.07 (3sf)
ans: 4.07mm
solving a proportion of the form x/a = b/c
To solve a proportion like x/a = b/c, you need to find the value of x that makes the two ratios equal. You can do this by cross-multiplying, which involves multiplying the numerator of one ratio by the denominator of the other ratio, and vice versa. We start by multiplying both sides by ac, giving us x = (ab)/c.
In summary, to solve a proportion of this form in just three steps, we multiply both sides by the product of the denominators, then simplify the resulting expression to get the value of x. To solve a proportion like x/a = b/c, you need to find the value of x that makes the two ratios equal. You can do this by cross-multiplying, which involves multiplying the numerator of one ratio by the denominator of the other ratio, and vice versa.
Following the cross-multiplying method, you multiply the two numbers that are diagonally opposite each other, then do the same for the remaining two numbers. In this case, multiply 'a' by 'c' and 'x' by 'b'. This results in the equation: x * b = a * c. To find the value of x, divide both sides of the equation by 'b'. This will give you the final equation: x = (a * c) / b. By plugging in the values for a, b, and c, you can easily solve for x and find the value that satisfies the proportion x/a = b/c.
To know more about ratios visit:-
https://brainly.com/question/28345307
#SPJ11
What is the value of x that makes the equation
3x - 4= 15 + 2x true?
You may show your work either by drawing on the
sketch pad
Answer:
\(\boxed {x = 19}\)
Step-by-step explanation:
Solve for the value of \(x\):
\(3x - 4 = 15 + 2x\)
-Take \(2x\) and subtract it from \(3x\):
\(3x - 4 - 2x = 15 + 2x - 2x\)
\(x - 4 = 15\)
-Add \(4\) to both sides:
\(x - 4 + 4 = 15 + 4\)
\(\boxed {x = 19}\)
So, the value of \(x\) is \(19\).
What is the solution of x+15(x−1)=1?
Answer:
x +15 + x -1 =1
x+x +15-1 =1
2x-14=1
2x-14-14=1-14
2x =-13
2x÷2=-13÷2
x=-6 and a half
simplify the following expression. 5.3x − 8.14 3.6x 9.8 a. -2.84x − 1.66 b. 8.9x 1.66 c. -2.84x 17.94 d. 8.9x 17.94
The simplified expression is (-187.584x + 287.6672) / 6.8, which is equivalent to option A: -2.84x - 1.66.
To simplify the expression 5.3x - 8.14 / 3.6x - 9.8, we can first simplify the division by finding a common denominator for the fractions.
The common denominator for 3.6x and 9.8 is 3.6x * 9.8 = 35.28x.
Next, we can rewrite the expression using the common denominator:
5.3x * (35.28x/35.28x) - 8.14 * (35.28x/35.28x) / 3.6x * (35.28x/35.28x) - 9.8 * (35.28x/35.28x)
Simplifying further, we get:
(5.3 * 35.28x^2 - 8.14 * 35.28x) / (3.6 * 35.28x - 9.8 * 35.28x)
Now, we can simplify the numerator:
(187.584x^2 - 287.6672x) / (-6.8x)
Factoring out an x from the numerator, we have:
x(187.584x - 287.6672) / (-6.8x)
Finally, we can cancel out the x terms:
(187.584x - 287.6672) / -6.8
Therefore, the simplified expression is (-187.584x + 287.6672) / 6.8, which is equivalent to option A: -2.84x - 1.66.
To know more about simplified expression refer here:
https://brainly.com/question/29076148
#SPJ11
Solve for b in the proportion. 2/b = 16/32
The value of b is 4 after solving the expression 2/b = 16/32, on cross multiplication, the answer is 4.
What is an expression?It is defined as the combination of constants and variables with mathematical operators.
It is given that:
The expression is:
2/b = 16/32
After cross multiplication:
32x2 = 16b
The arithmetic operation can be defined as the operation in which we do the addition of numbers, subtraction, multiplication, and division. It has a basic four operators that is +, -, ×, and ÷.
16b = 64
b = 64/16
b = 4
Thus, the value of b is 4 after solving the expression 2/b = 16/32, on cross multiplication, the answer is 4.
Learn more about the expression here:
brainly.com/question/14083225
#SPJ1
A closed half-plane is the solution of a linear inequality that comes close to the boundary line.
O True
O False
Answer:false
Step-by-step explanation:
Two integers between 60 where the square root lies
It is answer choice (C). 7 and 8!
Really hope this helped! brainliest would be very appreciated!
~HAPPY DAYZ~
5x−2(x+1)=1/4 find x pretty please
Answer:
Delaney solve for X by isolating it, or getting it by it self
Step-by-step explanation:
5x−2(x+1)=1/4 (distribute the 2)
5x-2x - 2 = 1/4 (subtract 2x from 5x)
3x -2 +2 = 1/4 +2 (add 2 to both sides)
3x = 2_1/4 (put 2 & 1/4 together)
3x = 9/4 (convert 2_1/4 to 9/4)
\(\frac{1}{3}\)*3*x = \(\frac{1}{3}\)*\(\frac{9}{4}\) ( mutilply both sides by 1/3)
X = 3/4 (cross multiply )
There you go
X = \(\frac{3}{4}\)
Answer:
\(\huge\boxed{\bf\:x = \frac{3}{4}}\)
Step-by-step explanation:
\(5x - 2(x+1)=\frac{1}{4}\)
Use the distributive property for 2(x + 1).
\(5x - 2x - 2= \frac{1}{4}\)
Subtract 2x from 5x.
\(3x - 2= \frac{1}{4}\)
Bring 2 to the right hand side of the equation .
\(3x = \frac{1}{4} + 2\)
Add 1/4 & 2.
\(3x = \frac{1}{4} + \frac{8}{4} \: \: [LCM = 4]\\3x = \frac{9}{4}\)
Now, bring 3 to the right hand side of the equation .
\(x = \frac{9}{4} \times \frac{1}{3}\\x = \frac{9}{12}\\\boxed{\bf\:x = \frac{3}{4}}\)
▬▬▬▬▬▬▬▬▬▬▬▬▬▬▬▬▬▬▬▬▬
Benjiro and Eren are playing Fortnite. Benjiro had 4 kills in 10 minutes. Eren had
3 kills in 8 minutes. If each player continues at the same rate, who will reach more kills in 40 minutes? How many kills will each player have? Explain you answer. *
Answer:
Benjiro will have more kills because 40 divided by 10 is 4 so 4 times 4 is 16 while 3 times 5 is 15
Step-by-step explanation:
Jackson has to have his car repaired. The mechanic tells him it will cost $75 for each hour of labor plus the cost of the parts. He discovers that the parts cost $165.
Part A: Write an equation that can be used to determine the cost of the repairs for any hours of labor.
Part B: How much would the repairs be if it took 3 hours?
Part C: What is the meaning of the slope in Part A?
Part D: What is the meaning of the y-intercept in Part A?
Answer:
a 75x+165=y
b 75(3)+165=390
c 75x is the cost increasing for each hour
d y-intercept=165, it cost at least 165 for the parts
Step-by-step explanation:
Quick help would be appreciated : )
The number of liters of fuel required for the vehicle to travel 1, 428 kilometers is 5. 67
What is inverse variation?Inverse variation can be defined as the relationship between two variables, so that if the value of one of the variable increases the value of the other variable decreases.
Example: If a varies inversely as b
This can be represented as;
a ∝ 1/b
Then,
ab = k
Where k is the proportionality constant
From the information given, we have that;
The number of kilometers = xThe liters of fuel = yx ∝ 1/ y
Then,
xy = k
Substitute the values
476 (17) = k
k = 8092
If x = 1428 and k = 8092
Let's determine the value of y
y = k/x
y = 8092/1428
y = 5. 67 liters
Hence, the value is 5. 67 liters
Learn more about inverse variation here:
https://brainly.com/question/13998680
#SPJ1
A class has 4 sections P, Q, R and S, with their average weights of the students in them are 45lb, 50lb, 55lb and 65lb, respectively. What is the maximum possible number of students in section R if there are 40 students in all sections combined and the average weight of the all students across all the sections is 55lb
Therefore , the solution of the given problem of median comes out to be R students still present (maximum) =35
Explain median.The number that precisely falls in the center of a sorted collection is the median value. An average or maximum probability metric is the difference between the initial and final 50% of the data. Different algorithms are employed to compute the average and mode depending on if you do have an unusual or perhaps even number of data points.
Here,
Median is 55
P :Q: R :S :average of 45 :50: 55 :65
The number of pupils within every section needs to be as low as possible
R is a mean
P differs by 1: 10 from the mean.
Q is 1.5 away from the mean.
Then S will just have three students, or three times five, to keep the population's mean constant.
R students still present (maximum):
=> 40 - 1- 1-3
=> 35
Therefore , the solution of the given problem of average comes out to be R students still present (maximum) =35
To know more about median , visit
brainly.com/question/28060453
#SPJ4
(-9,6) and (-3,2) help pls
(-9,6) and (-3,2) help pls
Answer : -12.8
Given y+6=10(x-9) What is the point used to write the equation?
Answer:
Point (9, -6)
General Formulas and Concepts:
Algebra I
Point-Slope Form: y - y₁ = m(x - x₁)
x₁ - x coordinate y₁ - y coordinate m - slopeStep-by-step explanation:
Step 1: Define
y + 6 = 10(x - 9)
Step 2: Break Function
Identify Parts
Slope m = 10
Point (9, -6)
Simplify sin2q cot2q.
Answer:
cos2q
Step-by-step explanation:
sin2q x cos2q/sin2q
= cos2q
(10 points) can someone help? :(
Answer:
Where is the Question?
Step-by-step explanation:
suppose a random sample of n = 64 measurements is selected from a population with mean µ = 65 and standard deviation η = 12. find the probability that x falls between 65.75 an 68.75
the probability that x falls between 65.75 and 68.75 is approximately 0.087 or 8.7%.
To find the probability that x falls between 65.75 and 68.75, we need to calculate the z-scores corresponding to these values and use the standard normal distribution.
The z-score is calculated using the formula:
z = (x - µ) / σ
where x is the value, µ is the population mean, and σ is the population standard deviation.
For 65.75:
z1 = (65.75 - 65) / 12 = 0.0625
For 68.75:
z2 = (68.75 - 65) / 12 = 0.2917
Using a standard normal distribution table or a calculator, we can find the probabilities corresponding to these z-scores.
P(0.0625 < z < 0.2917) = P(z < 0.2917) - P(z < 0.0625)
By looking up the values in the standard normal distribution table, we find that P(z < 0.2917) = 0.6159 and P(z < 0.0625) = 0.5289.
Therefore, P(0.0625 < z < 0.2917) = 0.6159 - 0.5289 = 0.087
Hence, the probability that x falls between 65.75 and 68.75 is approximately 0.087 or 8.7%.
To learn more about probability click here:brainly.com/question/31828911
#SPJ11
please answer asap (25 points)
only write down what I gotta put in the box and explanation
Answer:
12
Step-by-step explanation:
Use 5 for m
Use -3 for x
Use 7 for y
y =mx + b
7 = (5)(-3) + b
7 = -5 +b
12 = b
the matirial for one headband costs $0.95. Each headband is sold foe $3.80. What percentage of $0.95 is $3.80?
A study was done on the weights of different types of fish in a pond. A random sample of fish were caught and marked in order to ensure that none were weighed more than once. The sample contained 150 largemouth bass, of which 30% weighed more than 2 pounds. Which of the following conclusions is best supported by the sample data?
A) The majority of all fish in the pond weigh less than 2 pounds.
B) The average weight of all fish in the pond is approximately 2 pounds.
C) Approximately 30% of all fish in the pond weigh more than 2 pounds.
D) Approximately 30% of all largemouth bass in the pond weigh more than 2 pounds.
According to the sample data, the best conclusion supported by the sample data is that approximately 30% of all largemouth bass in the pond weigh more than 2 pounds. The answer is (D) Approximately 30% of all largemouth bass in the pond weigh more than 2 pounds.
The conclusion that is best supported by the sample data is that approximately 30% of all largemouth bass in the pond weigh more than 2 pounds, which is based on the fact that the sample contained 150 largemouth bass, of which 30% weighed more than 2 pounds.
Therefore, options A and C are incorrect. The sample data does not provide enough information to determine the average weight of all fish in the pond, so option B is also incorrect.
The correct answer is D) Approximately 30% of all largemouth bass in the pond weigh more than 2 pounds. This conclusion is supported by the sample data because the study specifically included a sample of 150 largemouth basses, of which 30% weighed more than 2 pounds. The other options are not supported by the sample data because the study does not include data on any other types of fish or the average weight of all fish in the pond.
To know more about the "sample data": https://brainly.com/question/24466382
#SPJ11
The cost of renting a canoe to use on River Y costs $30.The cost of renting a canoe to use on River Z costs $3 per hours plus a $12 deposit. The total cost, c, of renting a canoe on either river for n hours can be represented by an equation. How many Hours do you have to rent a canoe for the cost to be the same
Answer:
6 hours on river z is $30
Equation is 3c+12=30
C=6
In a statistical inference method you'll draw a conclusion in the end - you’ll _______ something - about the characteristics of what you're comparing.
In a statistical inference method, you'll draw a conclusion in the end - you'll infer something - about the characteristics of what you're comparing.
Statistical inference is a process used in statistics to draw conclusions or make predictions about a population based on a sample. It involves using data from a sample to make inferences about the larger population from which the sample is drawn.
To perform statistical inference, you typically follow these steps:
1. Formulate a research question or hypothesis: Start by identifying what you want to investigate or test. For example, you might want to determine if there is a significant difference in test scores between students who receive tutoring and those who do not.
2. Collect data: Gather relevant data through surveys, experiments, or other methods. In the example above, you would collect test scores from both the tutoring and non-tutoring groups.
3. Choose an appropriate statistical inference method: There are various statistical inference methods available, such as hypothesis testing, confidence intervals, and regression analysis. Select the method that best suits your research question and data.
4. Analyze the data: Apply the chosen statistical inference method to the collected data. This involves performing calculations and statistical tests to draw conclusions about the population based on the sample.
5. Draw conclusions: Based on the results of the analysis, draw conclusions about the characteristics of the population you are studying. In our example, you might conclude that students who receive tutoring have significantly higher test scores than those who do not.
In summary, in a statistical inference method, you use data from a sample to draw conclusions or make predictions about the characteristics of a larger population. This process involves formulating a research question, collecting data, selecting an appropriate inference method, analyzing the data, and drawing conclusions based on the results.
Learn more about inference:
https://brainly.com/question/3390566
#SPJ11